Need to resort to spherical wavefront to derive the LTs?

In summary: I think it's a bit of both. The Pythagorean theorem is important because it relates the spatial dimensions to the time dimension, which is crucial in deriving the Lorentz transformations. And the linearity of the transformations is also important because it allows us to simplify the problem by looking at just one spatial dimension. Overall, both elements are necessary for a complete and accurate derivation of the Lorentz transformations.
Physics news on Phys.org
  • #107
Saw said:
Thanks, but as stated in the text from me that you have quoted I am not interested (in this thread) in deriving the LTs and then noticing that they "preserve a particular quadratic form in spacetime", but acting the other way round: deriving the invariant ST interval and then looking for what transformations preserve it. In particular, I am interested in what assumptions are needed to make this ST interval derivation
I don’t think that your objection to @strangerep is valid. A derivation of the Lorentz transform and then a derivation of the spacetime interval is still a derivation of the spacetime interval.

At a maximum Einstein’s two postulates are necessary for deriving the ST interval. This is done by deriving the Lorentz transform and then showing the invariance of the interval. That forms a valid proof of the ST interval.

There are several valid “one postulate” derivations, but they all require one postulate plus experimental evidence to determine the value of the unknown parameter, c. So that is still two assumptions. I don’t think it can be derived in less.
 
Last edited:
  • #108
Dale said:
I don’t think that your objection to @strangerep is valid. A derivation of the Lorentz transform and then a derivation of the spacetime interval is still a derivation of the spacetime interval.
I think @Saw wants to be able to derive the LTs, ##\Lambda##, from ##\eta=\Lambda\eta\Lambda^T## given ##\eta##, rather than deriving ##\eta## given ##\Lambda##. Strangerep's approach is the latter. So what's needed is a thought experiment that leads to the interval/##\eta## without explicit dependence on the Lorentz transforms.
 
  • #109
Ibix said:
I think @Saw wants to be able to derive the LTs, ##\Lambda##, from ##\eta=\Lambda\eta\Lambda^T## given ##\eta##, rather than deriving ##\eta## given ##\Lambda##. Strangerep's approach is the latter. So what's needed is a thought experiment that leads to the interval/##\eta## without explicit dependence on the Lorentz transforms.
I think it is fairly pointless for you and I to argue about @Saw’s intentions. To me he seems to be focused on the number of assumptions, particularly implicit or hidden assumptions, required for deriving the spacetime interval. But it isn’t a good sign that more than 100 posts in this is a matter of confusion
 
  • #110
Ibix said:
I think @Saw wants to be able to derive the LTs, ##\Lambda##, from ##\eta=\Lambda\eta\Lambda^T## given ##\eta##, rather than deriving ##\eta## given ##\Lambda##. Strangerep's approach is the latter. So what's needed is a thought experiment that leads to the interval/##\eta## without explicit dependence on the Lorentz transforms.
Thanks, that is exactly what I want. And then calmly reflect on the assumptions inherent to such derivation of the ST interval. As simple as that. Shall we do it on the basis of Wheeler and Taylor's derivation, as you started to do? @strangerep suggested that Taylor and Wheeler's derivation may not be good enough, but I myself cannot think of a better one, so I would work on the basis of that one.

If so, would you agree, as a first step, that the steps of such derivation are as follows?

1671893481216.png
 
  • Sad
Likes Dale
  • #111
Dale said:
I don’t think that your objection to @strangerep is valid. A derivation of the Lorentz transform and then a derivation of the spacetime interval is still a derivation of the spacetime interval.
Talking about the didactically convincing "derivation" of some particular result in physics is of course always thinking about, which assumptions should be made in this derivation.

Einstein's choice was to assume the validity of Newton's Lex Prima, the special principle of relativity (existence of a (global) inertial frame) and the independence of the speed of light in vacuo on the velocity of the light source relative to the observer. Then, using (not so explicitly stated) also the assumption that space for any inertial observer is described as a Euclidean affine manifold (as in Newtonian mechanics). He derived the Lorentz transformation from using these postulates to synchronize clocks, at rest relative to each other and relative to an inertial frame, and how the description of space and time change for an observer moving with constant velocity relative to this inertial frame, thus defining also an inertial frame. Then he proved that also the Maxwell equations are Lorentz invariant, and the problem with the incompatibility of Maxwell theory with Galilei invariance and the observed absence of an absolute inertial frame, defined as the rest frame of some "aether". That's a physicist's derivation, i.e., giving a instrumental definition of how to synchronize clocks in such a way as to make the two postulates, which took from the features of Maxwell's equations under the assumption that they must obey the special principle of relativity, the very ones that are necessary to establish a new mathematical description of space and time given the absence of an absolute inertial reference frame.

Another derivation, that is much simpler mathematically, is to use Einstein's postulates together with the Euclidicity of space for any inertial observer and just use the invariance of the Minkowski quadratic form (which implies the invariance of the Minkowski bilinear form) to derive the Lorentz group (or rather the Poincare group by assuming space-time-translation symmetry too) as the corresponding symmetry group of the implied space-time model, i.e., Minkowski space as a 4D affine manifold with an indefinite fundamental from (Minkowski bilinear form) of signature (1,3) or equivalently (3,1). This has the advantage of great mathematical elegance but the disadvantage of being pretty abstract rather than physical as is Einstein's derivation.

Another variant of this more mathematically inclined pedagogics is to only use the special principle of relativity and the Euclidicity of space for any inertial observer to figure out what the possible spacetime symmetries are, and I found the answer pretty appealing when I first read about this approach: It comes out that there are only two possible spacetime symmetries (and thus spacetime models, because these can be derived from the symmetries), i.e., Galilei-Newton or Einstein-Minkowski spacetime.

It think, in fact, it's best to use both, the physics and mathematics approach to the "derivation of the Lorentz/Poincare transformation", because it has a higher chance of gaining a really deep understanding or the transformation from both the physical and the mathematical meaning.
Dale said:
At a maximum Einstein’s two postulates are necessary for deriving the ST interval. This is done by deriving the Lorentz transform and then showing the invariance of the interval. That forms a valid proof of the ST interval.

There are several valid “one postulate” derivations, but they all require one postulate plus experimental evidence to determine the value of the unknown parameter, c. So that is still two assumptions. I don’t think it can be derived in less.
Indeed, you need pretty constraining symmetry assumptions about time and space to arrive at the Minkowski spacetime of special relativity. You can, however, leave out the 2nd of Einstein's postulate and derive that there's only Galilei-Newton or Einstein-Minkowski spacetime left. Then you can leave it to the experimentalists figuring out which model is more accurate in describing Nature. The answer is well known since Maxwell: It's Einstein-Minkowski spacetime, and to a very high accuracy the limiting speed of the Minkowski space-time model is indeed the speed of em. waves in vacuo. The latter is an empirical conclusion from the determination of the upper limit of the photon mass, which is at ##m_{\gamma} < 10^{-18} \text{eV}##.
 
  • Like
Likes Dale
  • #112
Saw said:
that is exactly what I want.
Post 110. Just saying…
 
  • #113
vanhees71 said:
Another variant of this more mathematically inclined pedagogics is to only use the special principle of relativity and the Euclidicity of space for any inertial observer to figure out what the possible spacetime symmetries are, and I found the answer pretty appealing when I first read about this approach: It comes out that there are only two possible spacetime symmetries (and thus spacetime models, because these can be derived from the symmetries), i.e., Galilei-Newton or Einstein-Minkowski spacetime.
Adding to what I said in post 110, my intention is precisely to zoom on the assumptions that make us prefer the Einstein-Minkowski spacetime through the adoption of the so-called ST interval (the invariant Minkowski bilinear form, as you refer to it in the previous paragraph).
 
  • #114
Saw said:
my intention is precisely to zoom on the assumptions that make us prefer the Einstein-Minkowski spacetime through the adoption of the so-called ST interval
But that in no way prevents you from doing the derivation through the Lorentz transform. I think that you need to decide what you really want. More than 100 posts in and it seems to change every time we hear from you.

If you want to minimize assumptions then any of the two postulate derivations of the Lorentz transform or any of the one postulate derivations plus an experimentally determined invariant speed will get you to the spacetime interval.

If you want to derive the spacetime interval without the Lorentz transform then you may not be able to minimize the assumptions.
 
  • Like
Likes vanhees71 and PeterDonis
  • #115
Saw said:
Adding to what I said in post 110, my intention is precisely to zoom on the assumptions that make us prefer the Einstein-Minkowski spacetime through the adoption of the so-called ST interval (the invariant Minkowski bilinear form, as you refer to it in the previous paragraph).
I've just looked at this thread. I've no idea how you dragged it out to 110+ posts. It's crazy!
 
  • Like
Likes vanhees71
  • #116
PeroK said:
I've just looked at this thread. I've no idea how you dragged it out to 110+ posts. It's crazy!
Post dilation?
 
  • Haha
Likes vanhees71 and Dale
  • #117
Dale said:
If you want to derive the spacetime interval without the Lorentz transform then you may not be able to minimize the assumptions.
I never said that I want to minimize the assumptions!!!! I just said that I want to have a clear idea about which are needed and which are not and my concern is precisely that I fear that often too few are declared, as I have repeated plenty of times!!!. Please stop questioning what the object of the OP is, It is what Ibix said in his post 108 and I confirmed in post 110: deriving the ST interval (full stop, not LT) and, yes, of course, being clear on how (on the basis of which assumption this has been done). Is that so difficult to understand? It seems so, given how often I have to repeat it, but it is not so.
PeroK said:
I've just looked at this thread. I've no idea how you dragged it out to 110+ posts. It's crazy!
I am also quite tired that the object of the thread is constantly put into question. If anybody wants to discuss this simple thing, he/she is invited to stay. Whoever does not like it, thanks for your comments so far and goodbye. If anybody wants to close the thread, fair enough. Yet the truth is that the object is clear and I am making a good-faith attempt to discuss it.
 
  • #118
Saw said:
Shall we do it on the basis of Wheeler and Taylor's derivation, as you started to do?
There's nothing left to do after post #105 and the reference in @Sagittarius A-Star's #106 that fills in the setup for spacelike and null separated events, except fill in the value of the flight time of the light, which tells you that ##\Delta t=2l/\sqrt{c^2-v^2}## (where ##l## is the separation of the mirrors) and ##\Delta x=v\Delta t##. Those are general expressions valid in all frames moving perpendicular to the line between mirrors, and ##c^2\Delta t^2-\Delta x^2## is manifestly invariant. I already listed the assumptions used.
 
  • Like
Likes Motore and vanhees71
  • #119
Saw said:
I am also quite tired that the object of the thread is constantly put into question.
It keeps getting put into question because nobody but you seems to know what it is.

Saw said:
It is what Ibix said in his post 108 and I confirmed in post 110
So it took 110 posts to finally get what you say is a clear statement of what you want. If you are tired after all that, imagine how the rest of us feel.

But even then you throw us another change:

Saw said:
my intention is precisely to zoom on the assumptions that make us prefer the Einstein-Minkowski spacetime through the adoption of the so-called ST interval
What is this "make us prefer"? Where did "prefer" come into it? Prefer over what?

If you want to know what assumptions are required to derive the Minkowski interval (I have no idea why you keep saying "ST"--what does that stand for?), that can be done without any "preference" at all. It's just a straightforward question of logic, which has been answered.

If you want to know why we "prefer" the Minkowski interval over something else, the answer should be obvious: because it makes correct predictions. But that's a different question from the one you said you wanted answered in post #110.
 
  • Like
Likes vanhees71 and berkeman
  • #120
PeterDonis said:
I have no idea why you keep saying "ST"--what does that stand for?
Space Time, I assumed
 
  • Like
Likes vanhees71
  • #121
Saw said:
I never said that I want to minimize the assumptions!!!! I just said that I want to have a clear idea about which are needed and which are not
That is minimizing the assumptions. If an assumption is not needed then you can discard it and use fewer assumptions.

Saw said:
Please stop questioning what the object of the OP is, It is what Ibix said in his post 108 and I confirmed in post 110 … Yet the truth is that the object is clear
If a reader must wade through 110 posts to get a statement of purpose from the OP then the object of the thread is not clear. I am not questioning your good faith, but the outcome of this thread is a mess. The title and your OP are not focused on the interval but also on the Lorentz transform. The derivation you were first focused on was a derivation of the Lorentz transform. So in your mind you may now be certain of what you want, but that doesn’t make it true “that the object is clear” to the rest of us.

Saw said:
deriving the ST interval (full stop, not LT) and, yes, of course, being clear on how (on the basis of which assumption this has been done). Is that so difficult to understand?
You can derive the spacetime interval by first deriving the Lorentz transform and second showing that the form of the interval is invariant under the Lorentz transform. That is a perfectly legitimate derivation. You have rejected this legitimate approach. So, yes, it is difficult for me to understand.
 
  • Like
Likes vanhees71
  • #122
Dale said:
You can derive the spacetime interval by first deriving the Lorentz transform and second showing that the form of the interval is invariant under the Lorentz transform. That is a perfectly legitimate derivation. You have rejected this legitimate approach. So, yes, it is difficult for me to understand.
I will in the end agree that this is crazy... Come on, who said that that (first LT, later ST) is not a legitimate approach? Who has rejected that approach? I have just said that the other approach, which is also legitimate (first, ST, then LT), is the one that I am interested in, for whatever reason. This is the last time that I answer a comment about the object of the thread. If you want to discuss how to derive the *ST interval* and on which assumptions, few or many, welcome. Otherwise, I can understand that you prefer to drop out, but don't clutter the thread with more comments about the object of the thread, 'cause this makes it actually unfollowable.
 
  • #123
Saw said:
I will in the end agree that this is crazy...
Thread closed for Moderation...
 
  • Like
Likes Saw
  • #124
Saw said:
Come on, who said that that (first LT, later ST) is not a legitimate approach? Who has rejected that approach?
You did:
Saw said:
I am not interested (in this thread) in deriving the LTs and then noticing that they "preserve a particular quadratic form in spacetime"
Anyway, your question as finally clarified is already answered. So we will leave this thread closed.
 
  • Like
Likes vanhees71, weirdoguy, Saw and 1 other person

Similar threads

  • Special and General Relativity
Replies
7
Views
428
  • Special and General Relativity
Replies
1
Views
1K
  • Special and General Relativity
Replies
12
Views
1K
  • Special and General Relativity
2
Replies
36
Views
3K
  • Special and General Relativity
Replies
22
Views
1K
  • Introductory Physics Homework Help
Replies
10
Views
844
  • Special and General Relativity
Replies
33
Views
2K
  • Special and General Relativity
Replies
26
Views
369
  • Special and General Relativity
2
Replies
40
Views
2K
  • Special and General Relativity
Replies
25
Views
871
Back
Top